Cardio1 Flashcards Preview

STEP1 > Cardio1 > Flashcards

Flashcards in Cardio1 Deck (171)
Loading flashcards...
1
Q
A

Mitral valve stenosis

The high PCWP (i.e., left atrial pressure), and the large gradient between PCWP and the LVEDP, indicated a stenoic mitral valve. Also, a stenotic mitral valve would increased the resistance to flow into the left ventricle, which would cause blood to back up in the circuit. We see this manifested in the high pulmonary artery pressure

2
Q

What is the cause of this woman’s shortness of breath?

A

Pulmonary congestion.

The stenotic valve is increasing the resistance to flow from the left atrium to the ventricle, which is causing blood to back up in the circuit. This will lead to pulmonary congestion and increased pulmonary capillary pressure, leading to pulmonary edema and difficulty breathing.

We can also tell that pulmonary capillary wedge pressure is elevated because the pressures are elevated at the arterial and venous ends of the pulmonary circulation.

3
Q

Diastolic murmur best heard at the 5th left intercostal space at the midaxillary line

A

mitral valve stenosis

4
Q

What is the pulmonary capillary wedge pressure an estimate of?

A

left atrial pressure

5
Q

When would you hear a murmur due to mitral regurgitation?

A

during diastole

6
Q

What is the normal range for pulmonary capillary wedge pressure (PCWP)?

A

4-12 mmHg

7
Q

What is indicated by a PCWP that is significantly greater than the LVEDP?

A

This means that there is a pressure gradient across the mitral valve, which indicates stenosis

8
Q

What pressure changes would you see in someone with aortic stenosis?

A

Because the resistance to ejection of blood into the aorta is increased, you would see greatly increased left vantricular pressure. The systolic pressure in the aorta, however, would be normal

9
Q

What type of murmur is produced by aortic regurgitation? Where can it best be heard?

A

Blowing, high-pitched, decrescendo diastolic murmur, best heard along the lower left sternal border

10
Q

What is aortic regurgitation? How does this manifest clinically?

A

When blood flows back through the aortic valve into the left ventricle during diastole. This results in a blowing diastolic murmur, and a widening pulse pressure, sometimes greater than 100 mmHg

11
Q

Why is the systolic pressure increased in someone with aortic regurgitation?

A

The systolic pressure is increased because of an increased stroke volume, which is secondary to an increased LVEDV, all due to the aortic regurgitation.

12
Q

A 67 yo man comes to the physician for a physical exam required for insurance. During blood pressure measurement, systolic sounds are heard in the brachial artery with the cuff completely deflated. His BP is 170/80 mmHg. What is the most likely diagnosis? Why can these sounds be heard when the cuff is deflated?

A

This man likely has aortic regurgitation, evidenced by the large pulse pressure, and the systolic sounds heard with a deflated cuff.

In aortic regurg., the systolic pressure increases (the pulse pressure widens) because of increased SV. This very high volume and pressure results in turbulent flow through the brachial artery that can be heard even when the cuff pressure is zero.

13
Q

Where can aortic stenosis best be auscultated?

A

upper right sternal border

14
Q

What are the classic blood pressure findings in someone with aortic stenosis?

A

elevated LV peak systolic pressure, normal aortic systolic pressure = pressure gradient across aortic valve.

In aortic stenosis, there is increased resistance to blood flow through the aortic valve. In this case, the left ventricle must generate greater than normal pressure to push blood into the aorta (increased peak systolic pressure in th eleft ventricle), while the aortic systolic pressure will be normal.

15
Q

A 42 yo woman comes to the ED b/c of a 1-hr hx of crushing substernal chest pain. An EKG shows critical stenosis of a cardiac valve. It is determined the stenotic valve is the aortic valve. what will be heard on auscultation?

A

A crescendo, decrescendo systolic murmur with an ejection click

16
Q

Decrescendo diastolic murmur best heard at the lower left sternal border

A

aortic regurgitation

17
Q

Diastolic rumble best heard in the left lateral decubitus position.

A

Mitral stenosis

18
Q

Holosystolic murmur radiating to the apex

A

mitral regurgitation

19
Q

What might you hear on auscultation of a pericardial effusion? Why?

A

Muffled heart sounds, as the fluid surrounding the heart blocks the conduction of the heart sounds.

20
Q

A 70 yo man is brought to the ED b/c of a 1-month hx of worsening SOB. Auscultation shows a systolic murmur best heard at the upper right sternal border. CXR shows enlargement of the left vetnricle. Which of the following additional pathologic changes would most likely be associated with this condition?

(a) calcification of the aortic valve
(b) dilatation of ascending aorta
(c) dissection of ascending aorta
(d) rupture of aortic valve leaflets
(e) vegetations on the aortic valve

A

calcification of the aortic valve.

This is the most common cause of aortic stenosis in the elderly

21
Q

A 70 yo man is brought to the ED b/c of a 1-month hx of worsening SOB. Auscultation shows a systolic murmur best heard at the upper right sternal border. CXR shows enlargement of the left vetnricle. What is the diagnosis? What is the relevance/connection of the enlargement of the left ventricle.

A

This patient has aortic valve stenosis.

Aortic stenosis –> increased afterload –> ventricular hypertrophy –> subsequent dilatation of the left ventricle

22
Q

What is coarctation of the aorta?

A

A narrowing (stenosis) of the aorta, which increases the vessel resistance, and forces the heart to pump against a higher pressure.

23
Q

What is the difference in location of the coarctation of the aorta between the infant and adult forms?

A

In the infant form, the coarctation lies distal to the aortic arch, but before (proximal to) the PDA

In the adult form, the coarctation lies distal to (after) the aortic arch (distal to the left subclavian artery), and is not associated with the PDA

24
Q

With what genetic disorder is the infantile form of coarctatin of the aorta associated?

A

Turner Syndrome

25
Q

How does coarctation of the aorta present in adults?

A

Hypertension in the upper extremities, and hypotension in the lower extremities –> a strong radial pulse and a weak femoral pulse.

Classically, notching of the inferior margins of the ribs is seen on CXR.

Increased left ventricular wall thickness (hypertrophy in response to pumping against a higher pressure)

Decreased vessel wall thickness in arteries distal to obstruction

26
Q

A 3 month old girl is brought to the physician because of dyspnea, difficulty feeding, and poor weight gain. Physical exam shows a weak femoral pulse compared with the radial pulse. Pressure tracings from the thoracic and abdominal aorta are shown in the graph. Which of the following is most likely to be increased above normal values in this patient?

(a) abdominal wall thickness
(b) femoral artery wall thickness
(c) left ventricular wall thickness
(d) resting blood flow in leg muscles
(e) renal blood flow

A

left ventricular wall thickness

This infant has coarctation of the aorta, which increased the pressure against which the heart must pump, leading to left ventricular hypertrophy.

(The wall thickness of vessels distal to the coarctation will be decreased)

27
Q

A 25 yo man comes to the physician because of episodes of headache and light-headedness. He also has pain in his calves on walking that is relieved by rest. BP is elevated in the upper extremities and below normal in the lower limbs. Diagnosis? Why the headache and light-headedness?

A

Coarctation of the aorta.

In this condition, there is a narrowing of the aorta just distal to the left subclavian artery. This leads to increased blood flow and hypertension in vessels proximal to the stenosis (upper extremities), and hypotension in the lower extremities. The headache and light-headedness/dizziness are manifestations of hypertension

28
Q

A 25 yo man comes to the physician because of episodes of headache and light-headedness. He also has pain in his calves on walking that is relieved by rest. BP is elevated in the upper extremities and below normal in the lower limbs. Diagnosis? Why the calf pain that is relieved by rest?

A

Coarctation of the aorta.

In this condition, there is a narrowing of the aorta just distal to the left subclavian artery (aortic arch). This leads to decreased blood flow to and hypotension in the lower extremities. Hypotension can manifest with signs and symptoms of ischemia. This patient has claudication: pain in the legs when walking that is due to ischemia and relieved with rest.

29
Q

What is the classic X-ray finding in coarctation of the aorta? Why do we see this?

A

Notching of the interfior margins of the ribs.

This is due to the development and enlargement of collateral arteries, in particular, the intercostal arteries. The enlargement of the arteries, which run along the inferior margin of the ribs, causes notching of the ribs

30
Q

What is Eisenmenger syndrome?

A

A shift from a left-to-right shunt to a right-to-left shunt secondary to pulmonary hypertension

31
Q

A 26 yo man comes to the physician because of a chronic cough. He has smoked 1 pack of cigarettes per day for 10 years. He also gets frequent headaches and aches in his legs when he exercises. A CXR shows irregularities and scalloping on the undersurface of his ribs. What is the likely diagnosis?

A

Coarctation of the aorta

Scalloping, or notching, on the undersurface of the ribs is pathognomonic for this disease.

(The headaches and aches in his legs when exercising [i.e., claudication] are also consistent with upper extremity hypertension and lower extremity hypotension that is classically seen in this condition).

32
Q

Besides due to the narrowing of the aorta itself (and therefore increased blood flow to upper-extremity vessels), what is another cause of the hypertension seen in coarctation of the aorta?

A

Coarctation of the aorta also results in hypotension in the lower extremities. This means there will be reduced blood flow to the kidney, resulting in activation of the renin-angiotensin-aldosterone system, which will increased blood pressure.

33
Q

A previously healthy 74-yo woman is brought to the ED b/c of a fall 2 days ago. The woman tripped and fell at home, injuring her right leg, and has been unable to get out of bed since the injury because of pain. 2 hours ago, she suddenly became confused and disoriented. Temp: 37 degrees C; pulse 110/min; RR 32; BP 120/70. Pulse ox shows ox saturation of 80%; CXR shows no abnormalities. Which of these is the most likely diagnosis?

(a) acute cerebral hemorrhage
(b) acute cerebral infarction
(c) MI
(d) pulmonary infection
(e) pulmonary thromboembolism

A

Pulmonary Thromboembolism

Hip fracture and bed rest are classic risk factors for development of a PE.

A classic clinical presentation of PE is hypoxemia (ventilation/perfusion mismatch) despite normal CXR, tachycardia, and mental status changes in older patients.

34
Q

A previously healthy 74-yo woman is brought to the ED b/c of a fall 2 days ago. The woman tripped and fell at home, injuring her right leg, and has been unable to get out of bed since the injury because of pain. 2 hours ago, she suddenly became confused and disoriented. Temp: 37 degrees C; pulse 110/min; RR 32; BP 120/70. Pulse ox shows ox saturation of 80%; CXR shows no abnormalities. Why would we rule out cerebrovascular events (a and b)

(a) acute cerebral hemorrhage
(b) acute cerebral infarction
(c) MI
(d) pulmonary infection
(e) pulmonary thromboembolism

A

These could produce the mental status changes we see in the patient, but they would not directly cause the hypoxemia, unless the patient was hypoventilating (e.g., brainstem involvement)

35
Q

A previously healthy 74-yo woman is brought to the ED b/c of a fall 2 days ago. The woman tripped and fell at home, injuring her right leg, and has been unable to get out of bed since the injury because of pain. 2 hours ago, she suddenly became confused and disoriented. Temp: 37 degrees C; pulse 110/min; RR 32; BP 120/70. Pulse ox shows ox saturation of 80%; CXR shows no abnormalities. Why would we rule out infection?

(a) acute cerebral hemorrhage
(b) acute cerebral infarction
(c) MI
(d) pulmonary infection
(e) pulmonary thromboembolism

A

Pulmonary infection could produce mental status changes, tachycardia, and hypoxemia, but the onset of symptoms would be slower, the patient would likely be febrile, and the CXR would be abnormal

36
Q

What two processes/systems get activated in response to hemorrhage to return BP to normal?

A

(1) The baroreceptor reflex is stimulated, which increases sympathetic activity and decreases parasympathetic activity. This will increase HR and TPR
(2) Decreased blood flow to the kidney –> increased renin release, activating the renin-angiotensin system. This will lead to increased blood pressure and blood volume.

37
Q

How does the CNS monitor arterial blood pressure?

A

Via arterial baroreceptors located in the carotid sinus (innervated by CN IX) and the aortic arch (CN X)

38
Q

Which nerve innervated the baroreceptors in the carotid sinus? Which innervates those in the aortic arch?

A

Carotid sinus: CN 9

aortic arch: CN 10

39
Q

Between the baroreceptors in the carotid sinus and the aortic arch, is one group more important than the other in monitoring arterial blood pressure?

A

Quantitatively, the carotid sinus is more important

40
Q

Where in the CNS is the information from the carotid sinus and aortic arch sent to?

A

The nucleus solitarus in the medulla

41
Q

What is the effect of decreased BP on the arterial baroreceptors?

A

decreased BP –> decreased stretch of arterial walls –> decreased firing of baroreceptors –> decreased parasympathetic outflow + increased sympathetic outflow –> increased HR and BP

42
Q

What conducts the parasympathetic outflow in the baroreceptor reflex?

A

The vagus nerve, which decreases HR

43
Q

In the baroreceptor reflex, what is the affect of increased sympathetic outflow?

A

increased HR and BP

44
Q

What causes increased renin secretion?

A

(1) increased sympathetic activity
(2) The juxtaglomerular (JG) apparatus in the kidney monitors BP; decreased BP –> increased renin secretion

45
Q

What does renin do?

A

converts angiotensinogen to angiotensin I

46
Q

What does angiotensin II do?

A

(1) stimulated arteriolar vasoconstriction
(2) stimulates aldosterone release

47
Q

How does sympathetic activity lead to increased total peripheral constriction? (signalling molecules, receptors)

A

increased sympathetic outflow –> increased norepinephrine release from postganglionic neurons –> increased stimulation of alpha-1 adrenergic receptors –> vasoconstriction

48
Q

A 48 yo man is brought to the ED b/c of severe chest pain on exertion. Physical exam shows a 3/6 pansystolic murmur in the aorta with radiation to the carotids. Echocardiogram confirms the presence of aortic stenosis. There is increased blood flow through the coronary arteries. What would explain the increased blood flow?

A

Increased adenosine, which dilates the coronary arteries.

aortic stenosis –> increased afterload –> increased left ventricular pressure to eject blood –> increased work by left ventricle –> increased consumption of oxygen and ATP –> increased production of adenosine

In this way, coronary blood flow is coupled to, and increases in proportion to, the oxygen demand/consumption of the heart

49
Q

A 48 yo man is brought to the ED b/c of severe chest pain on exertion. Physical exam shows a 3/6 pansystolic murmur in the aorta with radiation to the carotids. Echocardiogram confirms the presence of aortic stenosis. There is increased blood flow through the coronary arteries.

If there is increased coronary blood flow, why is this man experiencing angina?

A

Although blood flow is increased in an attempt to increase oxygen delivery to the tissue, it is not enough to meet the excessive demand. This excessive demand is likely due to increased ventricular mass.

50
Q

A 60 yo man is admitted to the hospital b/c of a 2-day hx of SOB. His BP is 75/50 mmHg, and HR is 83. Phys. exam shows 3+ pitting edema of the lower extremities bilaterally. Labs show BUN is 62 mg/dL and serum creatinine is 3.0 mg/dL. A CXR shows cardiomegaly and perihylar infiltrates. What is the cause of his lab results?

A

prerenal azotemia

The SOB, low BP, caridomegaly, perihylar infiltrates, and pitting edema are all indicative of congestive heart failure. This will lead to decreased renal perfusion (we see his BP is low). Congestive heart failure is a common cause of prerenal aoztemia.

A BUN:Cr >20 is indicative of prerenal azotemia; in this patient, it is >20.

51
Q

What is an ateriovenous fistula?

A

An abnormal connection between an artery and a vein.

52
Q

How would CO output be affected if there is a large fistula that involves a major artery?

A

It would increase

The fistula means that there is an abnormal connection between an artery and vein. This would mean that oxygen concentration is decreased in the arterial blood, and increased in venous blood. To maintain oxygen delievery to the tissues, CO will increase (via an increase in HR and SV)

*The increase in CO is roughly equal to the blood flow through the fistula

53
Q

How would BP be affected by a large fistula involving a major artery?

A

BP and systemic vascular resistance would be decreased

The fistula provides a low-resistance pathway to shunt blood from the artery to the vein, decreasing BP and systemic vascular resistance.

54
Q

The solid line (A) shows a normal heart; The dashed line (B) shows a heart with valvular disease.

How does the work of the heart in patient be compare to that of patient A? How do you know?

A

The heart represented by B is doing increased work.

The area enclosed by the PV loop represents the work done by the heart when ejecting the stroke volume during the cardiac cycle. The area of loop B > the area of loop A

55
Q

How does the oxygen consumption of heart B compare to heart A?

A

The oxygen consumption of heart B is increased compared to A.

We can see from the area of the loops that the work done by B is greater than A, and oxygen consumption/demand is directly related to the work.

56
Q

The solid line (A) shows a normal heart; The dashed line (B) shows a heart with valvular disease.

Why is the work of patient B’s heart increased while the stroke volume is still the same?

A

This patient has a valvular disease that is increasing the afterload on the left ventricle, which must generate a higher systolic pressure than normal to eject the same volume of blood.

57
Q

What is the typical gross and histological appearance of a cardiac myxoma?

A

gross: forms a pedunculated mass, most commonly in the left atrium
histological: stellate mesenchymal cells admixed with inflammatory and epithelial cells.

(scattered mesenchymal cells in a prominent myxoid background)

58
Q

Why might a cardiac myxoma manifest in a person as syncope?

A

cardiac myxomas form a peduncuclated mass, most commonly in the left atrium. This mass can obstruct the mitral valve, inhibiting blood flow.

59
Q

A 70 yo woman collapses while shopping and dies of sudden cardiac arrest. Autopsy shows a solitary pedunculated mass within the left atrial chamber. Histologically, the cells are stellate mesenchymal cells admixed with inflammatory and endothelial cells. What is the most likely diagnosis?

A

cardiac myxoma.

The pedunculated mass in the left atrium, and the stellate mesenchymal cells admixed with inflammatory and endothelial cells, are classic gross and histological descriptions of a cardiac myxoma.

60
Q

A tumor is found in the heart. Histology shows densely-packed striated muscle. What type of tumor does this suggest?

A

Rhybdomyoma.

This is the most common primary cardiac tumor in children.

61
Q

tumor is found in the heart. Histology shows malignant glandular tissue. What type of tumor does this suggest?

A

Carcinoma, which is metastatic to the heart.

62
Q

What is the most common type of tumor found in the heart?

A

Metastatic tumors

63
Q

What cancers commonly send metastases to the heart?

A

lung cancer

breast cancer

lymphoma

melanoma

64
Q

A 55 yo woman come to the physician after several episodes of syncope. Phys exam shows a low-pitched “plopping” sound during diastole. 2D echocardiography shows a mass causing intermittent obstruction of the mitral valve. Diagnosis? What is causing the diastolic sound?

A

cardiac myxoma

this forms a pedunculated mass, most commonly in the left atrium. It can intermittently obstruct the mitral valve, resulting in the plopping sound (and in the syncope)

65
Q

In infective endocarditis, what are the vegetations on the valvular leaflets composed of?

A

firbrin, inflammatory cells (neutrophils), and colonies of bacteria

66
Q

What is the most common cause of infective endocarditis in IV drug users?

A

S. aureus

67
Q

What is the most commonly affected valve in endocarditis in IV drug users?

A

The tricuspid valve

68
Q

What is Libman-Sacks endocarditis?

A

non-infective (sterile) endocarditis that is associated with SLE (systemic lupus erythmatosus)

69
Q

What is the unique distribution of the vegetations seen in Libman-Sacks endocarditis?

A

The vegetations are present on both sides of the valve leaflets. (mitral valve)

70
Q

What is the underlying cause of non-bacterial thrombotic endocarditis?

A

(1) hypercoagulable state
(2) underlying adenocarcinoma

71
Q

How is the diagnosis of acute infective endocarditis confirmed?

A

With blood cultures, which are also necessary to determine the bacterial antibiotic sensitivity

72
Q

S. aureus is the most common etiologic agent of acute infective endocarditis in IV drug abusers. Why might this be the case?

A

Because s. aureus is commonly present on the skin.

IV needle can introduce skin organisms into venous system, which will then attack the tricuspid valve.

73
Q

What causitive agent is associated with subacute endocarditis in patients with prosthetic valves?

A

Staphylococcus epidermidis.

74
Q

What is the most common causative agent of endocarditis in previously damaged or congenitally abnormal valves?

A

streptococcus viridans

75
Q

S. aureus causes acute endocarditis, whereas S. epidermidis and S. viridans cause subacute endocarditis. Acute endocarditis is virulent and can lead to death in a few days; subacute endocarditis follows a more prolonged course and has a better prognosis.

A
76
Q

What causitive agents should be suspected in individuals with endocarditis with negative blood cultures?

A

HACEK organisms, which are normal commensals

Haemophilus

Actinobacillus

Cardiobacterium

Eikenella

Kingella

77
Q

What should we think of when we have a patient who is an IV drug user and has vegetations attached to the tricuspid valve?

A

infective endocarditis, s. aureus

78
Q

Is SLE associated with endocarditis

A

Yes, it can be. This is called Libman-Sacks endocarditis, which leads to the poroduction of aseptic vegetations on both the ventricular and atrial sides of the mitral valve.

79
Q

What valve is the most commonly involved in patients with endocarditis who previously had Rheumatic fever?

A

Mitral valve

80
Q

Tricuspid regurgitation will result in what type of murmur? where is is best heard?

A

Holosystolic murmur

left, lower sternal border

81
Q

What organism should be suspected in subacute bacterial endocarditis in elderly males who have had treatments of the urogential or biliary tract?

A

enterococci

82
Q

Streptococcus bovis is associated with endocarditis in patients with what co-morbidity?

A

Underlying colorectal carcinoma.

S. bovis endocarditis should prompt a colonoscopy to search for possible colon cancer.

83
Q

What should be suspected and empirically treated in a patient (typically young) who is an IV drug user with new-onset murmur/tricuspid regurgitation?

A

S. aureus

84
Q

Fetal alcohol syndrome is associated with what congential heart defect?

A

Ventricular septal defect

85
Q

What is the difference between the murmur heard in a ventricular septal defect (VSD) and in PDA (patent ductus arteriosus)?

A

Both murmurs are harsh-sounding murmurs.

However, the murmur associated with VSD will be holosystolic, whereas that associated with PDA is not holosystolic, but rather a continuous, machine-like murmur.

86
Q

An 18 month-old girl is brought to the physician by her parents because of frequent respiratory infections. Physical exam shows bilateral rales. CXR shows pulmonary congestion. Cardiac catheterization shows increased oxygen saturation in the right ventricle. Which of the following is the most likely finding on 2D echocardiography?

(a) Aortic insufficiency
(b) Mitral stenosis
(c) Patent ductus arteriosus
(d) Patent foramen ovale
(e) Ventricular septal defect

.

A

VSD

The left-to-right shunt that results from a VSD would lead to increased oxygen saturation in the right ventricle. It would also lead to increased blood volume and right ventricular pressure, leading to the pulmonary congestion we see in this patient.

87
Q

An 18 month-old girl is brought to the physician by her parents because of frequent respiratory infections. Physical exam shows bilateral rales. CXR shows pulmonary congestion. Cardiac catheterization shows increased oxygen saturation in the right ventricle. Why can we rule out (a) and (b)?

(a) Aortic insufficiency
(b) Mitral stenosis
(c) Patent ductus arteriosus
(d) Patent foramen ovale
(e) Ventricular septal defect

A

Neither of these would lead to increased oxygen saturation in the right ventricle.

88
Q

An 18 month-old girl is brought to the physician by her parents because of frequent respiratory infections. Physical exam shows bilateral rales. CXR shows pulmonary congestion. Cardiac catheterization shows increased oxygen saturation in the right ventricle. Why can we rule out (c)?

(a) Aortic insufficiency
(b) Mitral stenosis
(c) Patent ductus arteriosus
(d) Patent foramen ovale
(e) Ventricular septal defect

A

While PDA can result in a left-to-right shunt, this would not lead to increased oxygen saturation in the right ventricle.

The ductus arteriosus connects the pulmonary artery to the aorta, which allows blood to bypass the lungs during fetal development. While the blood going to the lungs in this shunt would have increased oxygen saturation, the blood in the right ventricle would not.

89
Q

What are the hemodynamic consequences of a VSD (ventricular septal defect)?

A

Formation of a left-to-right shunt.

The magnitude of the shunt will be proportional to the size of the defect.

90
Q

What is the difference between a patent foramen ovale and an atrial septal defect?

A

A patent foramen ovale refers to a residual, slit-like opening between the atria that is patent to a probe but of no hemodynamic significance.

An atrial septal defect is a much larger opening and has hemodynamic significance (left-to-right shunt).

91
Q

What is the hemodynamic significance of an atrial septal defect?

A

An ASD can result in a left-to-right shunt that will lead to increased oxygen saturation on the right side.

92
Q

An 18 month-old girl is brought to the physician by her parents because of frequent respiratory infections. Physical exam shows bilateral rales. CXR shows pulmonary congestion. Auscultation shows a harsh holosystolic murmur. Cardiac catheterization shows increased oxygen saturation in the right ventricle. Why can we rule out (d)?

(a) Aortic insufficiency
(b) Mitral stenosis
(c) Patent ductus arteriosus
(d) Atrial septal defect
(e) Ventricular septal defect

A

An ASD would result in increased oxygen saturation of the right ventricle. However, it would not produce a systolic murmur.

93
Q

An 18 month-old girl is brought to the physician by her parents because of frequent respiratory infections. Physical exam shows bilateral rales. CXR shows pulmonary congestion. Cardiac catheterization shows increased oxygen saturation in the right ventricle. Why can we rule out (d)?

(a) Aortic insufficiency
(b) Mitral stenosis
(c) Patent ductus arteriosus
(d) Patent foramen ovale
(e) Ventricular septal defect

A

A patent foramen ovale is patent to a probe but does not have hemodynamic significance. It would not result in any of the mentioned symptoms.

94
Q

What is classically heard on auscultation in a patient with patent ductus arteriosus?

A

A continuous, machine-like murmur.

95
Q

What are the hemodynamic and anatomical consquences of PDA?

A

Left-to-right shunt.

The left atrium and ventricle may be enlarged, and pulmonary hypertension may be observed, due to the increased blood flow caused by the shunt.

96
Q

In a patient with a patent ductus arteriosus (PDA), reversal of the shunt (Eisenmenger syndrom) may result in lower but not upper extremity cyanosis. Why?

A

The patent ductus arteriosus connects the pulmonary artery and aorta at a place on the aorta distal to the aortic arch. The blood supplying the lower extremities will mix with the deoxygenated blood from the right heart. However the blood to the upper extremities, which travels via braches off the aortic arch, will be unaffected.

97
Q

What can be heard on auscultation in a patient with an atrial septal defect?

A

The increased blood volume in the right heart due to the left-to-right shunt can result in:

(1) a systolic pulmonary ejection murmur
(2) a diastolic murmur (increased blood flow from atrium to ventricle)
(3) The second heart sound will be widely split.

98
Q

Gestational diabetes is associated with what congential cardiac defect?

A

transposition of the great arteries

99
Q

Would oxygen supplementation be very useful in a patient with transposition of the great arteries?

A

Slightly. In this condition, the aorta arises from the right ventricle, and the pulmonary trunk arises from the left ventricle. This results in pulmonary and systemic circuits that are completely separate.

This condition is only viable if there is shunt that allows blood from the two circuits to mix. Supplemental oxygen will only have an affect if a shunt is present.

100
Q

What can be used to maintain the patency of the ductus arteriosus in a patient with transposition of the great arteries?

A

PGE (prostaglandin E)

This compound maintains patency of the DA

101
Q

How would you treat a PDA?

A

You can give the patient indomethacin, which decreases PGE, resulting in PDA closure.

(PGE maintains the patency of the ductus arteriosus)

102
Q

A 26 yo man comes to the physician b/c of pain in his right leg. He has a Hx of trauma to the leg from a motorcycle accident 3 years ago. Phys exam shows a bruit over the popliteal space. A CT scan shows a popliteal arteriovenous fistula on the right. Which of the following would be increased in the femoral venous blood?

(a) CO2 content
(b) Hct
(c) O2 content
(d) plasma [Na]
(e) total protein concentration

A

O2 content

Some of the arterial blood is bypassing the tissues and mixing with the venous return. This will increase the oxygen saturation/content of the venous return.

103
Q

A 26 yo man comes to the physician b/c of pain in his right leg. He has a Hx of trauma to the leg from a motorcycle accident 3 years ago. Phys exam shows a bruit over the popliteal space. A CT scan shows a popliteal arteriovenous fistula on the right. How will (d) and (e) be affected?

(a) CO2 content
(b) Hct
(c) O2 content
(d) plasma [Na]
(e) total protein concentration

A

These values are normally similar in arterial and venous blood, and therefore should be unchanged by the fistula

104
Q

A 26 yo man comes to the physician b/c of pain in his right leg. He has a Hx of trauma to the leg from a motorcycle accident 3 years ago. Phys exam shows a bruit over the popliteal space. A CT scan shows a popliteal arteriovenous fistula on the right. How will (a) be affected?

(a) CO2 content
(b) Hct
(c) O2 content
(d) plasma [Na]
(e) total protein concentration

A

CO2 content should be decreased (closer to that seen in arterial blood) / lower than that seen in the opposite extremity. This is because arterial blood is mixing with the venous blood via the fistula.

105
Q

A 52 yo man comes to the physician b/c of progressive weakness and an inability to exercise. His symptoms have worsened over the last 2 years, and he has noticed swelling in his ankles. He has had no episodes of chest pain, arm pain, or jaw pain. He has a hx of elevated serum cholesterol. Which of the following is the most likely cause of his progressive symptoms?

(a) chronic ischemic heart disease
(b) myocardial infarction
(c) prinzmetal angina
(d) stable angina
(e) unstable angina

A

chronic ischemic heart disease

MI can be ruled out, because this typically presents with crushing substernal chest pain.

All forms of angina can be ruled out because these, by definition, involve paroxysmal chest pain (due to reversible myocardial ischemia)

106
Q

A 45 yo man presents at the ED b/c of chest pain that started 2 hours after shoveling; he states that he has had several episodes of chest pain during the past year when walking up stairs. ECG shows no abnormalities. In the ED, he is given nitroglycerin, which relieves the pain. Cardiac enzymes are within reference ranges now and a day later. Likely diagnosis?

(a) Prinzmetal angina
(b) stable angina
(c) subendocardial infarction
(d) transmural infarction
(e) unstable angina

A

Stable angina

Angina that occurs upon exertion in a person with a history of similar episodes is characteristic of stable angina. In stable angina, cardiac enzymes will also be normal, and ECG will show no abnormalities, or ST segment elevation due to subendocardial ischemia. Stable angina is also relieved with nitroglycerin.

**Normal enzymes rule out infarction

**only present at rest rules out prinzmetal and unstable angina. Unstable angina also is triggered with progressively smaller triggers.

107
Q

New-onset angina is, by definition, what kind of angina?

A

unstable angina

108
Q

How do the ECGs differ in angina versus infarction?

A

We will see ST-segment depression in stable and unstable angina, due to subendocardial ischemia.

ST-segment elevation is seen in myocardial infarction

ST-segement elevation is seen in Prinzmetal angina due to transmural ischemia

109
Q

Histologically, what classically distinguishes a premortem clot from a postmortem clot?

A

A premortem clot will contian lines of Zahn, which are laternating layers of platelets/fibrin and RBCs.

A premortem clot will also be characterized by attachment to the blood vessel wall

110
Q

What would we see on ECG in atrial fibrillation?

A

Absent P waves and an irregularly irregular QRS rhythm.

111
Q

What are marantic vegetations?

A

Small, non-infectious, acellular vegetations found on cardiac valve leaflets, often along lines of closure.

112
Q

With what pathologic process/disorder are marantic vegetations associated?

A

–non-bacterial thrombotic endocarditis

–Libman-Sacks endocarditis (associated with SLE)

113
Q

What does this image show? What does it indicate?

A

This shows lines of Zahn that are indicative of a premortem clot.

114
Q

What is the gold standard marker for establishing the diagnosis of a myocardial infarction?

A

Troponin I

This is the most sensitive and specific marker for MI

115
Q

How do troponin I levels change from the onset of myocardial infarction onward?

A

–levels rise 2-4 hours after infarction

–levels peak at 24 hours

–levels return to normal by 7-10 days

116
Q

What marker would be useful for detecting reinfarction that occurs days after an initial MI?

A

CK-MB (creatine kinase, isoform MB)

CK-MB levels rise 4-6 hours after infarction, peak at 24 hours, and return to normal by 72 hours. (recall: troponin I levels do not return to normal until 7-10 days after the initial MI, and would therefore still be elevated a few days after the initial MI).

117
Q

What is an abdominal aortic aneurysm, where does it usually arise?

A

An AAA is a balloon-like dilatation of the abdominal aorta; it usually arises below the renal arteries, but above the aortic bifurcation.

118
Q

What is the classic presentation of an abdominal aortic aneurysm?

A

A pulsitile abdominal mass

119
Q

What is the primary cause of an abdominal aortic aneurysm?

A

(severe) atherosclerosis

120
Q

A 57 yo man with type 2 diabetes comes to the physician for follow-up evaluation of his hypertension and hypercholesterolemia. Physical examination shows a pulsitile abdominal mass. An abdominal X-ray shows foci of calcium related to the abdominal mass. What is the likely diagnosis? Why?

A

Abdominal aortic aneurysm.

The pulsitile mass is a classic presentation.

The most common cause is atherosclerosis; he has 4 risk factors for atherosclerosis:

(1) male
(2) diabetic
(3) hypercholesterolemia
(4) hypertension

121
Q

A 57 yo man with type 2 diabetes comes to the physician for follow-up evaluation of his hypertension and hypercholesterolemia. Physical examination shows a pulsitile abdominal mass. An abdominal X-ray shows foci of calcium related to the abdominal mass. Why do we see foci of calcium related to the abdominal mass?

A

This patient has an abdominal aortic aneurysm, which is most frequently due to athersclerosis.

The foci of calcification occur within the athersclerotic plaques, and indicate severe atherosclerotic disease.

122
Q

How do fatty streaks appear histologically?

A

flat, yellow intimal streaks characterized by lipid-laden macrophages (foam cells).

123
Q

How does arteriolar density and total cross-sectional area change in response to hypertension?

A

it decreases

124
Q

How does capillary density and total cross-sectional area change in response to hypertension?

A

It decreases

125
Q

What happens to the arteriolar wall-to-lumen ratio in response to hypertension?

A

It increases

Untreated hypertension leads to hypertrophy of arteries and arterioles, causing the wall-to-lumen ratio to increase.

126
Q

If a patient had an abdominal aortic aneurysm, which of the labeled structures would be affected?

A

A

127
Q

What structures are indicated by (A) - (F)

A

A- aorta

B - descending colon

C - duodenum

D - inferior vena cava

E - kidney

F - ureter

128
Q

A 60 yo man comes to the ED b/c of sudden severe abdominal pain that radiates to his back. He has a hx of diabetes and HTN. Phys exam shows that he is tachycardic, tachypnic, and in acute distress. his BP is 76/40 mmHg. A large, pulsitile abdominal mass can be palpated, and an abdominal bruit is auscultated. Diagnosis? Why? What are his risk factors?

A

ruptured abdominal aortic aneurysm.

**abdominal pain that radiates to the back

**large, pulsitile abdominal mass

**abdominal bruit

**hemodynamic instability

**Hx of HTN and diabetes puts him at risk

129
Q

A 60 yo man comes to the ED b/c of sudden severe abdominal pain that radiates to his back. He has a hx of diabetes and HTN. Phys exam shows that he is tachycardic, tachypnic, and in acute distress. his BP is 76/40 mmHg. A large, pulsitile abdominal mass can be palpated, and an abdominal bruit is auscultated. What is indicated by this patient’s hemodynamic instability?

A

That the aortic aneurysm has ruptured

130
Q

A 68 yo man is brought to the ED b/c of sudden onset tearing chest pain. His chest pain awoke him from sleep and radiates to the back. The pain is not relieved by morphine. He has a Hx of hyperlipidemia and HTN. Exam shows a diastolic murmur along the left sternal border. The ECG is normal. A CXR shows widening of the mediastinum. Likely diagnosis and why?

A

Aortic dissection.

**sudden, severe, tearing chest pain that radiates to the back

**CXR shows widening of the mediastinum

**Diastolic murmur along left sternal border (aortic valve regurgitation)

**risk factors: hyperlipidemia and HTN

131
Q

A 68 yo man is brought to the ED b/c of sudden onset tearing chest pain. His chest pain awoke him from sleep and radiates to the back. The pain is not relieved by morphine. He has a Hx of hyperlipidemia and HTN. Exam shows a diastolic murmur along the left sternal border. The ECG is normal. A CXR shows widening of the mediastinum. Why would we rule out myocardial infarction in this case?

A

The ECG is normal.

In an MI, we would see ST-segment elevations

132
Q

A 68 yo man is brought to the ED b/c of sudden onset tearing chest pain. His chest pain awoke him from sleep and radiates to the back. The pain is not relieved by morphine. He has a Hx of hyperlipidemia and HTN. Exam shows a diastolic murmur along the left sternal border. The ECG is normal. A CXR shows widening of the mediastinum. Diagnosis? Why do we see a diastolic murmur?

A

Aortic dissection

In an aortic dissection, we have an intimal tear with dissection of blood through the aortic wall. That tear can go “backward” toward the heart. This can result in an widening of the aorta at its base, where it leaves the heart. This can cause a widening of the aortic valve that makes it inefficient, and allows for backflow during diastole.

133
Q

What are the 1st and 2nd most common causes of aortic dissection?

A

1 - HTN

2 - connective tissue disorders (e.g., Marfan’s, Ehlers-Danlos)

134
Q

Cystic medial necrosis is the histologic change associated with what cardiac disorder?

A

Aortic dissection, as seen in someone with a connective tissue disorder (Marfan, Ehlers-Danlos)

135
Q

A 50 yo woman comes to the ED b/c of pelvic pain. Phys exam shows several nodules within the endometirum and a diagnosis of uterine leiomyomas is made. The patient undergoes a total abdominal hysterectomy with bilateral salpingo-oophorectomy. A CT of the pelvis in a normal individual is shown. What structure is the arrow pointing to?

A

ovary

136
Q

What is the venous drainage of the ovaries?

A

Right ovary: pampiniform plexus –> gonadal (ovarian) vein –> inferior vena cava

Left ovary: pampiniform plexus –> gonadal (ovarian) vein –> renal vein –> IVC

137
Q

Name three ACE inhibitors

A

(1) Captopril
(2) Enalapril
(3) Lisinopril

138
Q

What do ACE inhibitors do?

A

Reduce levels of angiotensin II by inhibiting ACE (angiotensin converting enzyme)

This leads to a decrease in aldosterone levels

*recommended for the treatment of HTN and CHF

139
Q

What is Amplodipine?

A

A dihydropyridine calcium channel blocker indicated for the treatment of mild to moderate HTN and angina.

amlodipine selectively blocks calcium influx across membranes of smooth and cardiac muscle cells. The effect is greater in smooth muscle –> will cause arterial vasodilation without negative ionotropic effect.

140
Q

What is atenolol?

A

A beta-1 antagonist used for the treatment of HTN

141
Q

On an ECG, what is the T wave associated with?

A

Ventricular repolarization

142
Q

What part of an ECG corresponds to the absolute refractory period?

A

The beginning of the QRS complex to the apex of the T wave

143
Q

What part of an ECG corresponds to the relative refractory period?

A

The last half of the T wave

144
Q

What is tenting of the T wave on ECG indicative of?

A

hyperkalemia

This is because potassium is responsible for the repolarization of cardiac cells

145
Q

What are the clinical manifestations of hypernatremia?

A

nausea

vomiting

confusion

delirium

irritability

muscle twitching

146
Q

How will hypercalcemia manifest on an ECG?

A

Shortened QT interval

shortened or absent ST segment

147
Q

How will increased serum Mg2+ (hypermagnesemia) manifest on ECG?

A

prolonged PR and QT intervals on ECG

148
Q

What are the clinical manifestations of hypermagnesemia?

A

hypotension and respiratory depression

149
Q

Severe cell damage, such as severe burns, can lead to what serum ion changes?

A

This can lead to hyperkalemia, because potassium is one of the major ions present inside cells.

150
Q

S. bovis is a group D streptococci whose cell wall is composed of what?

A

glycerol teichoic acid

151
Q

What is represented by (A) the positively-sloped graph, (B) the negatively-sloped graph, and (C) the intersection of (B) and the x-axis

A

A - cardiac output curve

B - venous return curve

C - mean systemic filling pressure

152
Q

What is represented by the intersection of the curves?

A

In the steady state, the CO and the venous return must be equal to each other. The intersection of the graphs shows the CO, venous return, and right atrial pressure in the steady state.

153
Q

Point A shows the CO in a normal individual when the right atrial pressure is 0 mmHg. Point B represents the patient’s CO and right atrial pressure (RAP). Which of the following best explains these findings?

(a) arteriovenous fistula
(b) exercise
(c) impaired left ventricular contractility
(d) narcotic overdose
(e) severe hemorrhage

A

arteriovenous fistula

(b), (c), and (d) would all move the cardiac output curve. Because this curve doesn’t change, these can be ruled out. Severe hemorrhage (e) would decrease the mean systemic filling pressure (shift the intersection of the venous return curve with the x-axis to the left), and so can be rule out.

154
Q

How would spinal anesthesia effect the curves in the figure?

A

Spinal anesthesia would block the sympathetic nervous system. This would lead to systemic vasodilation –> decrease in the mean systemic filling pressure and a shift of the venous return point of x-axis intersection to the left. The decrease in sympathetic tone would also decrease resistance to venous return, causing the slope of the venous return curve to become steeper.

Finally, decreased sympathetic tone will lead to a decrease in the contractility of the heart, and cause the CO curve to shift down.

155
Q

When is an S3 heart sound heard? What creates this sound? What pathologic condition(s) is it indicative of?

A

Heard during diastole just after S2, as a result of blood oscillating back and forth between the walls of the ventricles.

It is indicative of heart failure (CHF) and volume overload

156
Q

Besides the location of auscultation, how can you differentiate between a right-sided S3 and a left-sided S3?

A

The right-sided S3 will increase with inspiration, while the left-sided S3 will increase with expiration.

157
Q

When do you hear an S4 heart sound? What is its cause?

A

Occurs just after atrial contraction at the end of diastole, just before S1.

It is the result of blood

158
Q

In what patients would you hear an S4 heart sound?

A

You hear it in patients with conditions that increase resistance to ventricular filling:

–hypertrophy

–HTN

–aotric stenosis

–cardiomyopathy

–coronary artery disease

159
Q

A 62 yo woman is brought to the ED by her husband b/c of SOB. She has a hx of type 2 DM, hyperlipidemia, and MI 2 yrs ago. Which of the following findings on physical exam is the most specific finding for CHF in this patient?

(a) a prominent S3
(b) a soft S4
(c) ascites
(d) pedal edema
(e) pulmonary rales

A

a prominent S3, which is classically indicative of heart failure or volume overload.

pedal edema and pulmonary rales can be see in CHF, but are also associated with non-cardiac disorders, such as hepatic cirrhosis.

160
Q

During an investigational study, two individuals volunteer for evaluation in a cardiovascular lab. When compared to the pressure-volume diagram labeled control, pressure-volume loop A most likely relfects which of the following changes?

(a) increased contractility
(b) increased end systolic pressure
(c) increased PCWP
(d) increased total peripheral resistance
(e) left ventricular dilatation

A

left ventricular dilatation

The LVEDPs in the control and in patient A are about the same, but the end diastolic volume in A is much greater than in the control. The only way for the pressures to be the same with different volumes is for the ventricle to dilate, which will increase its compliance.

161
Q

How does the end-systolic pressure and volume compare between the two loops?

A

The end systolic pressures are the same for both loops; the end systolic volume is greater for loop A than for the control.

162
Q

How does the contractility of the heart compare between the control and patient A?

A

The contractility is greater for the control than loop A. Contractility is given by the slope of the line drawn from the origin to the point of end systolic pressure:

163
Q

How does the PCWP compare between the two loops?

A

The PCWP is often used to approximate the LV end diastolic pressure, which is the same in these two loops.

164
Q

How does total peripheral resistance compare between these two loops?

A

Total peripheral resistance is lower in loop A than in the control.

Arterial diastolic pressure (DP) is typically proportional to total peripheral resistance; arterial DP is indicated by the pressure at which the aortic valve opens

165
Q

What is the effect of nitroglycerin on blood pressure and venous return?

A

It leads to a decreased in arterial blood pressure and a decrease in venous return.

Nitroglycerin dilates both arteries and veins. The dilation of veins –> decreased venous return.

Dilation of arteries and arterioles–the major resistance vessels–reduces arterial blood pressure.

Also, a decrease in venous return –> decrease in LVEDV –> a decrease in stroke volume –> decrease in CO –> a decrease in blood pressure.

166
Q

How does nitroglycerin affect LVEDV?

A

it decreases LVEDV

LVEDV = end systolic volume + filling volume per beat

End systolic volume is decreased due to decreased blood pressure, because there is less resistance to ejection.

Filling volume = venous return/HR

Nitroglycerin dilates arteries and veins. Dilation of veins –> reduction of venous pressure –> decreased venous return.

**Both decreased end systolic volume and decreased fillinf volume reduce LVEDV.

167
Q

Curve A shows a normal heart; curve B shows a heart with aortic valve disease. How does coronary blood flow during diastole in curve B compare to curve A?

A

It is increased.

The heart in curve B is doing increased work, as signified by the increased area in the PV loop. This increase in work increases the oxygen demand of the heart. Because 70% of the arterial blood oxygen is typically extracted by the heart, additional oxygen to the heart must be supplied by increased blood flow; increased extraction is limited. Also, the increased systolic pressure means that the coronary vessels are compressed more than normal, leading to decreased systolic blood flow through the coronary arteries. Decreased systolic blood flow + increased oxygen consumption and demand –> greatly increased blood flow during systole.

168
Q

How does coronary artery oxygen content compare between curves A and B?

A

It would be the same.

The myocardial oxygen consumption does not affect the oxygen content of the arterial blood.

The oxygen content of the coronary venous blood would, however, be decreased in curve B–the heart that is doing greater work.

169
Q

How does coronary blood flow during systole in loop B compare to loop A?

A

It is decreased.

The peak systolic pressure is greater in loop B, which compresses the coronary vessels even more than normal, causing blood flow to decrease.

170
Q

A 68 yo woman comes to the physician b/c of difficulty breathing. She has a past hx of angina pectoris. Physical exam of the lungs shows bilateral rales, jugular venous distension, and a positive hepatojugular reflux. THere is bilateral pedal edema up to the knees. Auscultation reveals an S3 best heard at the apex. An increase in which of the following best explains the swelling in the patient’s legs?

(a) interstitial colloid pressure
(b) lymphatic flow
(c) plasma colloid osmotic pressure
(d) right atrial pressure
(e) stroke volume

A

increased right atrial pressure

This patient has CHF, as evidenced by the S3, bilateral rales (pulmonary congestion), juglar venous distension and peripheral edema.

CHF –> reduced CO –> hypoperfusion of the kidneys –> activation of the RAAS –> fluid retention –> volume overload.

Increased right atrial pressure –> decreased venous return. In a volume-overloaded state, this will lead to increased movement of water into the interstium due to the increased hydrostatic pressure in the venous system –> edema = swelling in the patient’s legs.

171
Q

What is a positive hepatojugular reflux a marker for?

A

right heart dysfunction